Simple Proof of Weierstass Approximation Theorem?

Click For Summary
The discussion revolves around finding a continuous function G that matches g(x) on the specified domain D while being continuous everywhere. The user references a previous problem that established the existence of a polynomial sequence converging uniformly to any continuous function. They express uncertainty about how to explicitly determine G(x) after proving g(x) is continuous and suggest using limits and L'Hospital's rule for handling indeterminate forms. Additionally, they inquire about the application of the Weierstrass Approximation Theorem in this context. The conversation emphasizes the connection between polynomial approximation and continuity in function construction.
toddlinsley79
Messages
6
Reaction score
0

Homework Statement


Let D={x in the set of real numbers: -3<x<3, x does not equal 0,1,2} and define g(x)=(cosx-1)/x + (x3-2x2-x+2)/(x2-3x+2) on D. Find G:R→R such that G is continuous everywhere and G(x)=g(x) when x is in set D.


Homework Equations





The Attempt at a Solution



From a past homework problem I know how to prove that, for any continuous f:R→R, there exists a sequence (pn) of polynomials such that pn converges uniformly to f on any given bounded subset of R.
So after I show that g(x) is continuous and that a sequence of polynomials that converges uniformly to g exists, how do I actually find the function G(x)?
 
Physics news on Phys.org
Use the definition of continuity: g(0) = lim(x->0) g(x) = 1 etc. (The limits of the form 0/0 are to be tackled with L'Hospital's rule).
You might want to have a look at Riemann's theorem on removable singularities to see how this is done in general.
P.S. - Where is the Weirstrass Approximation Thm. called for?
 
Question: A clock's minute hand has length 4 and its hour hand has length 3. What is the distance between the tips at the moment when it is increasing most rapidly?(Putnam Exam Question) Answer: Making assumption that both the hands moves at constant angular velocities, the answer is ## \sqrt{7} .## But don't you think this assumption is somewhat doubtful and wrong?

Similar threads

Replies
3
Views
2K
  • · Replies 2 ·
Replies
2
Views
2K
Replies
3
Views
2K
  • · Replies 11 ·
Replies
11
Views
2K
  • · Replies 1 ·
Replies
1
Views
2K
  • · Replies 1 ·
Replies
1
Views
2K
  • · Replies 4 ·
Replies
4
Views
3K
  • · Replies 4 ·
Replies
4
Views
2K
  • · Replies 8 ·
Replies
8
Views
4K
  • · Replies 10 ·
Replies
10
Views
3K